You are on page 1of 42
M, Se. (Physics) Examination 2013 PHY 421 (Classical Blectrodynamies) rier n your ow word fora pructcae, The marks om the right-hand side denote the fll marks forthe ‘especie question. | Marks: 50 ‘Time: 2 Hoare ‘Answer Question no. 1 and any three from the rst 1. Answer any five 4x5 (a) Find the transformation propestes of the electric field the magnetic fed B and the Poynting vector under tine reversal transformation (0) A sudden burt of current 1(t) = n5(e),Alws through an infinite straight wire. Find the ‘vector potential due to this eurent ata distance fam the wit (6) TE and B are perpendicular in an inertial rane, show that they wil remain perpenieslar in all comoving frames, (4) Calculate the frst non-zero mulipele moment tensor for system of charges which are acranged as follows: q(¢/2,0/2),~g(0/2,~a/2), a(-a/2,~0/2) and -q(—0/2,0/2). (6) Starting fom an expression for the vector potential created by a given current density, show ‘that the dipole term ean be written as by mxr i vwhete [ser aot |s the magnetic dipole moment of the cutent dstebutio, (9 Indicate how sausage instability disrupts a pinched plasma column and indicate how it ean ‘be overcome by a longitudinal magnetic Fld (2) A charge +0 is distributed along the = axis rom —a to +4 0 thatthe charge density at = is siven by (2) = gpz. Show thatthe eloctrostate potential ata point is given by EF, apenrrtcan (b) The symmetric tensor 6% is given by PF = RPS + LAP where F§? isthe clactromagneti fed tensor. Obtain the various components of O in tems of three-varnbles. Write down th st of equations =aPY in terms of thre- quantities and comment onthe results. Here J denotes the current density 4+ G42) 0,8" 4. (a) Explain the principle of magnet bottle for plana confinement, What aguetic mir? (0) Stcting rm the action S'= = fdr (m-+a A!) of a charge g (with rst mas) moving Inn dcromget fly, ad he equation af motion. Hee «the vit fe Whar othe generalised momentum?” Explain the presence Of Sed dependent tem de xpreson of generlaed momentum (4H) + Han) (a) Beetially newt yz plan casas a vie dependent but uniform surface current JQ) twitched on at tine = 0. Show that th retarded vector potential ata height ato the Hoc, is ; 3 [soe (6) Calealate the electric and magnetic feds (6) Caleulate the power radiated by unit area of the current cacrying plane ata time, Express ‘his In ST units Ale 449404) (®) Wirite down the relations among the components of electric and magnetic felds in one inertia frame S, which is moving with respect to another inertial frame S' with a velocity 9 along common direction (b) Using the above relations, find the magnetic feld of « uniformly moving charge Q, with ‘velocity J along the direction. (6) Argue from dimensional ground why the magnetic feld of an accelerated charge falls off ax ata large distance 34542 6, (a) Magnetic field due to an accolrated charge, at a large distance from the charge is related to the electric field via B = # x B. Find an expression forthe angular distribution of radiated power. () Using the result derived in (a), cleaate the frequency spectrutn of radiated energy when an uniformly moving charge suddenly collides with an infinitely heavy scatterer (6) Assuu energy of a photon with frequency w 10 be fs, show that, in the abowe case the mumber of photons that would be emitted in the frequency range 0 ~+ w+ de i inverecly proportional tow B4542 M. Se. (Physics) 2nd Semester Examination 2013 PHY 422 (Quantum Mechanics 11) Jnsuer i your oun words as for as practicable, The marks om the right-hand margin indicate the fall mark for the question, lu saris: 50 ‘Time: 2 Hours ‘Answer Question No. 1 and any thee from the rest 1 Answer any five questions: 4x5=20 () Show that the quantum mechanical probability density obtained from the Klein-Goréon uation Is uot always positive, 4 () State and prove the adiabatic theorem in quantum mechatics, 3 (6) Using the party transformation ofa Dita spinor ¥ (no derivation needed) find the behaviour of PY and Pye¥ under party 242 (a) Consider a fee electron in uniform magnetic eld in the erection. With symmetric gauge choice, show that pe and Late constants of motion, On what factors does the degeneracy of the energy levels of sucha system depend? Ht (6) The amplitude for a two particle seatering proces is given as f(0) = a+ beosd (when the particles are diferent). Ifthe two particles are identical calculate the cross section at & scat- ‘ering angle of if tho particles ae () spines bosons and (i) unpolarised spin § fermions. 2+2 (0) Write down the Golden rule for constant transition rate in fist order time dependent peru ‘tion theory. Use this to obtain the expression of difereatial scattering crass setion according tothe fist Born approximation WS (a) Derive Thomas Reiche Kuh sum rule for a quantum system. Suppose the ocilstor strength 1" far t= 2p tation of byeoge atom i 042, what can you say abut other transitions? (b) Consider a particle bound to a fixed center by a spercally symmetric potential V(r). Show = wor (sB) (2) | for alls states, ground and excited, Verity the above relation for the ground state of « 3d harmonic osiiatar. () Explain clearly why the following matrix element is vanishing or non-vanishing (n= 2,1 = 1m =O] pe|n = 2,1 = 0,m = 0), where [nlm stands forthe energy eigenket of a ot-telativistic hydrogen atom with spin igoote. (2H) (342)4(041) (0) Show, using the aaticommuutation relations saised by the a (i =1,2,8) and 8 matrices ofthe Dirac equation, that they are tracles and evensdimensionl, (b) Defining ©, = —iazay and cyctic permutations, establish the relation (E.A)(x.B) ‘AB +:3.(A x B), where A and Bi are two ordinary vectors (242\46 . (a) Bxplain the importance of partial waves in scattering theory. Show that for short ranged potenti the -th partial wave phase shift at low energies behaves as 42", where k= ‘ith being the energy ofthe particle (b) For hard sphere scattering, show that the total scattering cross section for $ wave is four times the geometric cross-section. Explain briefly the fact that the phase shifts with { #0 ean be ignored in such a case (245)4(241) vi - (a) Two inertial frames are related by 2" = 2". The corresponding Dirac wave-functions aze connected by (2) = S{a)¥(2). Using the properties of $(a) for continuous Lorentz ‘transformations show that (W'@) = W(2)¥(2) () Sketch the potential V(r) = Voln(r/a) (Vo and a are constants). Show the turning point(s) inthe graph. Using the appropriate WKB approximation for {= 0 case, obtain tho allowed nergy levels of a particle inthe above potential. Is thee any interesting feature associated with the energy levels? Explain 5 + (1/241/24341) (2) A general rotation can be parametrized in terms of successive rotations through the Euler angles 0, 6,7 as follows: Ra, 8,1) Express (j,m'|R(a,,9)|j,m) in ‘terms of the Euler angles and j,m,m. (b) For a spherical tensor operators Tf where (—k < q < h) calculate [J2,T{]. Hence obtain FETE, m)) 5+ (342) / y M. Se, (Physles) 3rd Semester Examination 2013 PHY 512 (Statistical Mechanics) Answer in your oun words as far as practicable. The marks on the righthand margin indicate the full ‘marks for the question. AUl symbols have their usual meanings Full Marks : 50 Time: 2 Hours Group A Answer any three 1. (a) In the Mayer Ursll cluster expansion method to evaluate the partition function of a non-ides! ‘28s with 12 particles a term appears as constant ffushirfesfaofoardrisonneodria raw the corresponding graph and find {my). Show that it satisfies 5 n. ats] (©) Consider an ensemble of fermions at sero temperature ina volume V. Find the number density of particles in terms of the chemical potential. Show that Fy x n°, where Fy i the nero Point pressure and nis the particle mimber density es) 2. (a) In the Mayer Ure caste expansion the partition function can be written as tay (V2 ann) = {oes (me) Fad the grand canonical patton function. If wa write the equation of sate ax PV Sem (2) -Lam(Z) show that 0) =, and a2 = Be (b) Consider a Bose-Einstein gas with 1 = internal degeneracy 2. Show that in the ultrarelativistc limit (E = pe). ‘The gas has Bev 4 Pv = S(t) You may use {5° B45 = Ff 3] 3. (a) Show that the canonical partition function for a single free particle in a volume V is V/A* where 2 the thermal wavelength to) (b) For a quantum Bose gas the grand part nz = Syl (1~ 80°F) on function (2) ean be written as 3. What are the restrictions on 4? ji, Find the mean number of particles No in the ground state 4H, Show that there exists a temperature Tz above which No/N’ vanishes and for T+ T: osx (1~(7/1.)*"") uszs4) 4. (a) Consider a physical system described in a 3-dimensional space. Probabilities to observe the system inthe states (:) a(!) a(2) are 1/2,1/3 and 1/6 respectively. Find the density matrix, p, ofthe system. Show that Tg? < Tre= 5 (©) A system has two energy levels 0 and ‘Those san be oocupied by apinles fermions with temperature T and chemical potential jhe fermion are now intersting, ‘Write down the Partition funeton and find the average ocupation sumer ofthe state with energy. os9) Group B Anneer eny two 5. (a) Consider a canonical system consisting of sim partic in an external magnetic fl and a temperature Find ond (0) Determine the ctl exponent forthe flowing function as +0: J (©) Define critical exponents: What do you mean by unversality cas? ackes By Ha244) 6. (a) Por a one dimensional Ising system of IV spins in feld h, determine the partition function in terms of the eigenvalues of the matrix ne eo! 0-8) (©) Show that only the larger eigenvalue will contribute to the free energy: (©) Further show that at 2e0 magnetic feld the magnetization is er forall temperatures (+542) 7. (a) Express the n time joint probability density functim py of @ Stochastic Proces for a single variable €() in terms of an time conditional probability density function and an (n ~1) time joint probability density function (©) Show that fora stationary Markov process Palins it tno forn>2 (6) Calculate py. for completely sandom process e543) oa \ | M. Se. (Phys) and nd Semener Examination PHY'42 (Chase Becodynsiey [Answer in your um words as for {or as practicable. The marks onthe righ hand sde denote th fll mars forthe respectice question, pull Marks: 50 ‘Time: 2 Hours Answer Question no. 1 and any three fr 1. Answer any five any fi AG (©) (State Poynting’ theorem and write dowa its mathematica integral) form. (i) Starting from | ne tbone ial tna Rei ete sken WECg ye anme ethe tal eto ‘magnetic energy density and Sis the Poynting vector. Show that thie san euation of contin () Expression of the magnetic vector potential due toa magnetic dipole is giver by a= §B 52. (i) Show that the magnetic field due this dipole is given by B(?) = §@ 3 [a(ai.A)* ~ (G)Show that the interaction energy of two magnet dipoles separate ya distance ris ven by ons (6) Calculate electric and magnetic fields fom g(t, 5) = ~azyz and A(t, 2) = at (ysi+ 23) +208) ais a constant. Give arguments in support of your res (@) What should be the minimum energy of ravon (with rest mass 100 MeV) to emit Cerenkov radiation when it pases through water with RE? (©) Ror an eectromagnati ware prnueting i Lore rane, = B fin another Lorentz frame B= 31-+5) +24. Find the magnitude aod possible di the second frame of reference. (f) Use the argument of o-vnriance along with Coulomb's law of eletrestatis to find moving charge with velocity 3 (o) Find the charge and current densities that xive ssc the potentials aia — Hy aA) 1 (in the RHEL system). tions of Ein A of uniformly 2 gra (Bilet= lel? # for | E and if =a and 2 = b are the two turning points, Bnd out the barrier penotration probability using WKB approximation, _ (b) Verify using the Lorentz trasformation property of the Dirac wave-function that the parity transformation commutes with a ratation while it anticommutes with a translation. 4 (6) A particle is confined in a one-dimensional box between z= L. If the particle is in the ground state of the system, ealeulate the probability that i€ willbe in the ground state f the wall at 2 = 0 is suddenly moved to x = ~L: : (a) Obtain an expression for the spatial translation operator in one-dimensional quantum me ‘chanics, Under what conditions will it be a symmetry operation and what conservaton Taw wi) follow? 242 (6) Calculate the differential cross section in the frst order Born approximation for a potential given by = vo) (0) The amplitude fora two partic setterng proces is ven as (0) = a+ beost@ when the (sane diferent. Tithe two paris ar detcal wnpalarized spin-1 bosons calculate the Cross section at a scattering angle of 45° i 2. (a) Using first-order time dependent perturbation theory, obtain an expression forthe probebilty on arora iniual state i (0 final state J for a time-dependent perturbation V(0) {b) A one-dimensional simple harmonic osilator in its round sate. A perturbation Ve) oes aeeatehad on att = 0. What isthe probability of finding the system in the fist xc eae ee enn ine in the fist order Born approximation? What js the corresponding cit ne second excited state? (Given 2 = yA[TEmuy(a + a!) whete a and a! are the a ened creation operators, expectively. Other symbols carry their wsual meanings) a4 5 (a) Stating ftom the Dirac equation clulat the magnetic moment of» spn-} partes and show that it corresponds to a g-factor equal to 2 (8) Verify that the Dirae 7 matrices sntity {7°} = 29"T- (644) 4. (a) Two particles y and have reat energies 3.1. and 3.7 GeV, respectively, ‘hew aff tow (Riativiatc ct bound states, e being the charm quark ina linear potential V = Vo + kr. if Mow, seth O and n= 1 bound states of zero orbital engular momentum, predict the mave of "the m= 2 state with zero orbital angular momentum. {b) Show that for a scattering from a spherical potential, the total cross seetion can be written let + asin where 5; isthe phase-shift for the th partial wave 446 5. (a) Verify U = exp(insd) = cos ¢ + iys sing. Check U (2x4) = (Ep) Ut ana v (ps) = (HA). (0) Prove eatin scattering, phase sit is negative fr «potenti toe poatve fora potential that is attractive everywher at is repulsive everywhere, (243) +5 6. (a) Write down the components of « vector operator 4 in the spherical basis in terms of its cartesian components, Verify that they satisly the properties of tensor operator TZ with ku (b) Consider the sealar product of two vestor operators A and . Verify that A. operator. sealer (242) +6 'M. Se. (Physics) Sr Semestor Examination 2014 PHY 512 (Statistical Mechanics) | uate: 50 Tine: 2 ou Group A Joswer any five (Note : y = e*7) Consider an ideal Bose gas confine Ina region of area A in two dimensions. Expres the number of perils inthe excited state, Ne, andthe ground state, No, i terms of y, Tend and show that the system doesnot exhibit Bose-Einstein condensation unless T+ 0. 6 [Consider an ensemble of fermions at zero temperature in volume V. Show that Py an. Where ‘isthe sero point pressure and ni the particle number densi. 6) Starting from the relations NIV = aya) show that PV INET = 5% a4(09/)"* Consider classical gas of hard spheres of diameter ¢. When a particle is added tothe system of 1 particles, the volume avallable tothe new particle is not V but somewhat ls. Assuming that No! ce V, determine how the number of microstates (N,V, depends on V. Also show that as result of thi, V inthe gas law PV = NkaT gts replaced by (VB) where 8s equal to four times the actual space ocewpind hy the sphere 1s) Inthe theory of paramagnetism, the Hamiltonian can be written as Hai where jis the magnetic moment and His the magnetic feld, Treating the eystem classically find the magnetic moment ata temperature T. cl Fea quantum Bos gas te grand partition fenton (2) can be weten a ~¥,ln(1- #9) (e) Find the mean number of particles Np in the ground state, nz 10. n 2 (Hoe cor «cae wh 4 = 00 the ahr et (5 = pe) ad wth agency 2. Show at (B= pe) and with nny PV = SEED). You may we fo? BE = Fh Group B bea Answer any five In the mean-field theory of the Tsing model, show that the magnetization is determined by the expression m= tanh (qm +18 ‘where his the applied field ‘Obtain the graphical solution ofthis self-consistent equation at T., below T. and above T. 3) ‘The Hamiltonian forthe one dimensional Ising model i given by H=-LJoisies Using the transfer matrix method determine the partition function, 8 Consider the Hamiltonian of one-dimensional Ising system in afield. Derive the recursion relations for the parameters using teaspace Renormalization Group transformation. 6 Do the folowing funetious have crtieal exponents? If ye, determine them 4(j=A-Be $= abet A,B are constants. 8 (Show that for Ising model of interacting sin} paris, mean Bel heory gs pajad et {i) Wat are the casaleitics of «mate oe toes wt oppee 0 & onrequlibrium system? a (i Stave Onsager’ regression hypothe {i) Show that the correlation C(t) betwee ast 400, - 1 Buctuntions of «thermodynamical variable goes to x2 M. Sc. (Physics) Examination 2015 PHY 421 (Classical Electrodynamics) Anseern your own words es far as practice, The marks on the righthand side denote the ull mark or the respective question, Full Marks: 50 ‘Time: 2 Hours Answer Question mo. 1 and any three from the rest 1, Answer any five: 4x5 (@) Find the values of electric field (E) and magnetic feld intensity (1) on the surface of a ‘ite of length ! and diameter 2a carrying current I. The potential diference between the two ends of the wire is Vg. Compute the Poynting vector and show that It represents flow of energy into the wie (©) Using the Maxwel stress tensor, show thatthe magnitude of the net force on the ‘northern’ hemisphere of «uniformly charged sli sphere of radius and total charge Qi given by (©) Given, Fy Po" o E? — BY and FG oc BLB, where Gis the dual tensor to FH. (@) Explain why the electromagnetic field Lagrangiaz eannot have a term proportional to Fo (Gi) Given the values of Band B in an inertial frame, show that only the fed whose ‘magnitude is smaller can be made zero in another inertial kame, and that too only ifthe fields are perpendicular to start with (@) A point charge is performing SHM along the X-axis with a time period T. Argue that at a large distance from the charge, power radiated is proportional to T+ (©) An electron is released from rest and falls under the influence of gravity. In the fist ‘centimeter, what faction ofthe potential energy lest is radiated away? Given 14 Sr 10TW/A? and you ean assunie me = 10" Kg and g = 10 m/s. (0 A point charge is moving with uniform velocity J = fi. Show that the ratio of the ragnitudes of electric fel (at points onthe Kass and equidistant fom the charge) at the let and right ofthe charge is ( 2. (a) Suppose there are two sets of potentials (V4) and (V', 2), which correspond to the same slectrie and magnetic elds E and 6. What are the relaticns between the two potentials (6) Establish the diferential equations in terms of scalar potential V and vector potential A for charge density p and current density j (6) Write down the condition for Coulomb gauge. Hence show that V?V = £ and explain the consequence of solution of V in Special theory of Relativity. (4) Establish the Maxwell's equation © x 5 = yo) + soci under (i) Coulomb gauge and (i) Lorente gauge 242 +(142) 43 3 («) Consider the relativistic motion ofa charged particle of ret mass mo, velocity 7 and propa: zation veetor& and charge ein the feld ofa plane electromagnetic wave. Show that the quantity HE is « constant of motion Ae yas ft pane electromagnetic wave gen by = ot — EF wae Oe ively. Hence show aa (Ta ps tion vert mec. He hw ht edie a anu ne 6 es 1B = ea (E+ YES A] etn owe te lect fe for ie poled etromageetic wae Cee oe : io andy neu 7~U 0 oi (pansy mt er pti Ht hag tse ie pee ee LIE! ela Gate he ovr ined y the wit ofthe i Poe a orcs tnt thonognnn eal ens. be Cm Maco in te ree hacia ay om ae ie age ‘of mass m, was moving with uniform speed u and has been stopped to rest oa SR after being sattered by an infinitely heavy scatterer. Starting fom the ex Fostion for the angular distribution of radiated power (by an accleratd/deceated charge) ee Secale the wavelength ttn of ante ergy the es {b) Show that an accelerated charge must radiate to conserve energy and momentum. 73 (a) A carge @ (o) Bhetrc charge is glued on aplastic ring of radius aso that the ine charge density isin ‘The rng is spinning around an axis passing through its centre and perpendicular to ts plate with time period T. Calculate the vector potential due to the spinning ring a its centre (6) A charge 4 of mass m moves in a circular orbit of radius R with constant speed » .. What force one must exert on this (apart from the centripetal force) in order to have a stable obit? ou M. Se. (Physics) 2nd Semester Examination 2015 PHY 422 (Quantum Mechanics 11) Masks: 50 pl ‘Time: ¢ Hous Answer Question No, 1 and any three from the rest. 1, Answer any five questions: oe (0) Starting Som the Disc sgn ba he pe bias the popes ih the oan 8 mati sk Snes tat ac compos the Dire mre funn ein the ie Gator ent (©) Quantize the energy levels ofa bouncing ball. (V= mgz for #>0;V=c0 forz <0.) 4 (o) Find the probability that a tritium atom in the Is state after decay willbe in the Is state of He Given that the wae fanction of hydrogen atoms with nace care Zen the ts E a-tr v= where a isa constant 4 (4) A system was originally in a state i ofa slowly varying time-dependent Hamiltonian. Find ‘out the probability that it would be inthe state k (where By # E;) at time 7: 4 (6) Show that rotations in three- dimensions form an SO(8) group. 4 (f) Fora system of two identical particles which have no mutual interaction what are the common cigenstates of the Hamiltonian aad the permutation operator and what are the eigenvalues? If {tweak mutual interaction between the identical particles is introduced, how does it affect the energy of the above states? 22 2. (a) Fora particle in none dimensional potentatV(z) show that the the solution of the Senger uation in the WKB approximation is given ax 1 ole) = Apes emt [Hey wwhene (a!) = /BRATE= VM is the local wave vector. (@) Considering the hyperfine Interaction, the 1s state of hydrogen atom spits inte two levels corresponding to F = O and 1 where F ‘and the sfies e,p refer to decttons and rotons, respectively Ifthe atom was initially In the F = 1 state, calculate the probability that i wil bein the F = O state inthe distant future when a time dependent magnetic feld B, = Boexp(-t/7) i applied at = 0. (6) An electron i in the lowest state ofa 1-dimensiona infinite square well. At ¢= 0, a uniform ‘weak elect eld B is applied in the z dretion fr time T. Find using rst order perturbation theory the probability that the electron will be in the first excited state after the perturbation is removed 443 ee 2) nah ca of ei tan 8 cpg cnn oan een ee ee eel Ao eur ee A ar ere eomcid by We} = STAIN). Show tnt ac eee nn 545 Dirac cid by Dirac equation reqizes that SH(a)y*S( eg ee ee ed on tes a 7 setae ai Tuk) cv > +f pte Te ee ‘where the symbols have thelr usual meanings In the frst order Born approximation, the scattering amplitude fora particle of mass m off « spherically symmetric potential Vp) is given by sat = 228 [8-H ene where the symbols have their usual meanings. Find out the differential watterng cros section (2(0) for the Yukawa potential a vw ‘Obtain the Rutherford cross section formula as lit of the above result sean ‘5. (a) For the Dirac + matrices show that 7°94? = 9 for = 0,1,2,8. Use the properien ofthe 75 tmatrix (derivation not required) and this result to show that 790"? =O" where OF = >, (&) In the interaction pleture, the evolution operator for the system ket is given by fete fate fo eave) ita) Using this show that for time independent potential, the probability f transition is given by ve0)= SH 2 < flim >< mivii>f an | =) oun Fcsmis + {in the second order, where the symbols have their usual meanings. (aes) +5 6. (a) Denes symmetry transformation. Show that such a transformation must be ether wiry or aatuniary. (b) The quantity dpa) = (j,m'je%42/¥|j,m) >. Calculate d/2,(a). (6) Define a spherical tensor operator of rank k. Using the definition express the components of ‘spherical tensor of rank 1, XY (q = 41,0), in terms of the cartesian components of or % 1 XE 2,0), in terms ofthe cre comments 85, M. Se. (Physics) 3rd Semester Examination PHY 512 (Statistical Mechanics) ae jarks Full Mi be ‘Time : 2 hours Group A. Answer any three 1, (a) Starting from the following relations for the free Bose gas PLAT = gspalu)/®, N/V = gyralu)/® show that fe nEa (2) ‘Find the first two coeficients a; and dp, (AM the symbols have their usual meaning) (5 (b) Consider a system of extreme relativistic gas consisting of IV particles moving in one ‘dimension. Show that the partition funetion is given by Qntts7) = 2e(B))" L being the space available in one dimension. 6) 2. (a) Show that for a system in canonical ensemble (26)) = # [P* (By +r] 18) (b) A system has two energy levels 0 ond E. ‘Ths can be occured Dy legit with ‘icvesature 7 and chemical potential ‘The fermions aro non-interacting, ts down tempertiton function and id the average occuation numberof the state with exky IB. You do not have to consider the spin factor, [243] 3. (@) Consider an ideal Bose gas for which exp(—A(6— 1) << 1, 90 that one can use the expansion eres, Show thatthe introal energy #0 85 B= perv (pt) a SO (5) (0) Consider a physical system described in a $imensional Hilbert space. Probabiics to ‘observe the system inthe states (:) al) and } respectively, ind the density matrix pof the sytem. Show that Tr(p!) < 1 Tro 1 44. (0) Cousider an ensemble of fermions at zeo temperature in s volume V. Show that Py oc 1°, whore Pi the zeo point pressure and n isthe particle number density. 6 () Consider « microcanonical ensemble of N interacting particles where each particle can have two energy levels and e. The sate with energy cis doubly degenerat<. The total energy ofthe system is U |, Find the numberof mieostates in tems of U1 and « 4 Find the temperature and show that aeltirra ees] Group B Answer any two 5. (a) Stow thatthe Ising model Haniltonian under mean fld approximation inthe presence of an external maghotic field takes the fom , ~so(gden + 8) (b) Show that the mean field transition temperature T.= Jake and susceptibility Few (44(949)] 6. (0) For one dimensional Ising model, show that un asformatior ‘enormaied parameter an be expraned og nt SS" RO ae Ph = eh cosh(2In+ hn) cosh(2J~ ha) 0h = cos(2Jn + hp) coth(2Jn— ha) cosh* (hn) 2m & cosh(2In-+ hy) cosh(2J— hn) conta) given b= 2 (b) Determine the fixed points of the system, Show the :rajectories in parameter space [+940 7. (a) Show that fora stationary Markov process [fiver - sv] v6) : PalGataifacitesionif forn 22. (©) Caleuate pa for 0 completely random proces, (6) Find out the critical exponent forthe following funstions where ¢ denotes the deviation from the eitial point. i.) = 2504 + 0056-4 ii $() = APPPUL+ 05). ferze(2e2), M. Se. (Physics) 2nd Semester Examination 2016 PHY 422 (Quantum Mechanics II) tnover in your own words as for as practicable. The marks on the right-hand margin indicate the {full marks forthe question. asks: 50 ‘Time: 2 Hours ‘Answer Question No. 1 and any tire from the rest 1. Anower any five questions: 4x52 (a) What properties mast be satisfied by «transformation to be considered symmetry opera- tion? Show that asymmetry transformation mnt be wary or anthunitray 2 () Fora bound state establish the Bor Sommerfeld quantization condition fom WKB approx imation 4 (6) A particle is confined in a one dimensional box beween x = —[ to x = Land isin the round state. If suddenly one barier is moved from z= L to x = 0, find out the probability ‘hat the system wil bein ts new ground state. 4 (@) The ground state of the hydrogen stom splits into two lls due to hyperfine interaction ‘between the electron opin de and the proton spin 5 The two levels ocur for F = 0.1 when P= a+ with an energy difeence AE between then. A weak time dependent magnetic ld {B= By! i applied on the atom att 0. Find ont che probability that the atom wil be inthe a1 state ot time tie was oginaly i the F =O sate in (e) The Dae 0 and 8 matics satisfy (01.4) Stow that a and 9 are traclos matics i {or} =O, ands? (5 =1,2.9. (0) Por two-body interaction potential the satering amplitude behaves as (0) = 2+ boos? @ ta the cm frame, Iftwo denial fermions interact with his pte eleuiate (=) hen the two fermions are in (a) sin singleton 8 () spn triples state 32 (0) Cale te energy of hen sate ofthe ho tom sng WA apprimaion (0) Fora spberaly mec watering pte, ehh hat the atic crs sexon can ponents oa = ED (a+ in? wire the mbes ave the woul earings 6 2. () Aare with energy (> 0 isnt set potential given by V(D) = Ar=r) Fd othe as si he =O tate (0) A satnary exces i the ound ste of te Haiti H = gua fh mice Ba Bi. At a perturbing field V = gua A - Sexp(—t/r)/h is applied, when BF = BY and As, ‘pce B, Find out the probblty thatthe mstom wit not bein the ground mae at time when the perturbatlon odes ay. {e) Envi wy in ow every astern from ite ng plata it a sles to oer ‘ho fist few pati! waves aed? (a) Verity Paty? = a I= bs" ete previo rel to abow Pott? = 0 {0) For an fiona Loree trasfomaton 2" = (+o) (uy = 01,33) the Dine oe on nari na (2!) = S{u)¥(2) where S(a) = 1~ fayya™. ‘Determine the aa aeeston of B(x) © Wz? nod check tat W2)¥(2) remains invariant (242)+(64+1) Sov tnt th in on nro cis 72 nn ur te pr en fas [fasta vw 35 ® whore 7 is the time ordering operator and ater sytals have their usual meanings. 5 +5 (0 Stow tow rotation can be repo in tem SU) is {(b) From the definition of a tensor operator Tf calealate J {Tilivm >} and Je (TfL. >}, si jim an sng meta i 1 conser te sate 8 = [Pflu-1 > 4778 41>), whe YH akon em Oa ee co ants mometum egenate wih j= 1. Cabte Js > a8 Sle >: 4+ (ae) HD) M. Sc. (Physics) Examination 2016 PHY 421 (Classical Electrodynamics) Anscrin your 01m wor Jar as practical. The marks on the righthand mde deote the fl mark for the sl Macks: 50 Time: 2 Hours Answer Question no. 1 and any three frm the rest 1. Aner any ve as (0) Esplin why elcromagnetic ld has nly two ronerse dere of oaation in ite of ing 1 sector Bi ving componente You can chore ether Lorene or Cl aig ome our ee v () tnstatanoos potion of charge moving onthe x — y pane oe by 2) = 8+ Man Ui) =e Cacite the dition of the mage Bld Bly an nerve ting ot «pit (0.0) < (©) An exponentially decaying pois charge (() = gp) at ret atthe pint (0). What {hese potenti an wor pote Af yan cheer a he pot 0.) who movin og the 3 xn with aeocy vow Att = 0. ober wa (028) 22 (8) A somty moving chang (0 <<) of ot asm sujet oan ec eld = Bo int. Wit down the eqton of ation ofthis cage aking ou he ration reaction re Stow that on at ec. he adnton rato nen ene et v3 (©) Using the Mest ree ener sho hat tenet reo the norte hegre unborn charged aid pee of din and tea arg Qs iven ty = ch ie ‘ (0) Bxpronin of magic wector pots de toa magi dle en by A= fy (m=) {0 Show that the magus ell det hs diple men by Be) = fy (ene m {U) Show thatthe intercon ener of wo tngntc ipa (m ng) earned by datas ¢ de piven by U = iy om.ma 3m) 9 2/7) a 2. (a) Write down the diterential equations concering salar potential (V) and vector potential (A) under Coulomb gauge condition. Interpret the equation of V from Special theory of Relativit (©) Using the above diferential equations establish AAR op = onde Here, ju = - 20 SY Ve, O/le—e| and jr = BV «Vx fee le 0/le #1 (e) Hence establish the Maxwell’ equations for charge density (p) and current density () under Coulomb gauge (s2p443 4.) A orl pla plc wader ci ld () (with oe dy open ely) ad any dane tdci on), Siow tnt te ylation br pnt» ey ind) _B Ma) Gea where A a the mass of the fon and ws = €1/M. (©) Consider the relativistic motion of « charge particle of charge, rest mass rm, ‘propagation vector k in the fed of plane electromagnetic wave. The quantity § = velocity v and es ie 6 ™ he plane etree ve en B= a constant of motion and the phase (6) of ‘tr rxpecively, Show that the differential equa pee sant Seine ety at postion ot aia Ogee Sh peo (aE (c) Now suppone the sect fl or a neal pared tromagnetic wave Propagating logy Oe ne i aa eer tbe et sector le Sston Hee as ‘Eutions of the set of equations for x,y and =. Seas (q) Starting from the canonical dfition of energy momentum tensor, calculte the sme for tay ‘dynamics deseribed by ng + An a (0) Check whether the above expresion for TO? is gauge invariant, What kind of term ove mst ad to make it gauge invariant? Show thet 7°4 in its gauge inveriant form i aces () How does conservation of 79? imply that an accelerated charge must radiate energy? Be(uszeaye (a) Scalar potential (4) of « uniformly moving charge, e, with velocity Bi given by ot ino eR R= r= ¥' and #’ is the postion vector of the charge at time t— Rye Calculate £(e,u), Fourier transform of eletiefeld (ryt) show that at r >> 1, there i part in ‘Stctrie fed which yries as A. Why can this radiation term be non-zero only when the hare moving inside a refractive medium ? (b) Electrically neutral y~ = plane caries a time dependent but uniform surface current J()y switched Gear tie C= 0. Show that the rtarded vector potential at a height = above the plane ney [08 ‘Calculate the power radiated by wait area of the current earying plane at time f (1a) + 9) (6) Blectic field due to a slowly moving accelerated charge (e) is B= gafqeBt x (Rx) Use (8 ‘Glatt the freuen spectrum of edited energy when sorely mig charg es caliel 8 | fa eavy scatterer at {= fo so that its speod is being changing as we" for & >to (b) Calculate the minimum wave-length ofthe emitted ration (in terms of ee» an He | Imvof the charge) inthe above case.” You tay find the folowing integral handy ia your cds Sede 28) | () Explain radiated by an z i Eble] llr ie pole (oscillating with frequener ©) 57 a ‘iii aan a Som, 2017 M.Sc. (Physies) Examination 2017 PHY 421 (Classical Electrodynamics) ul Marks: 60 Answer Question no. 1 nd aty thre from the rest 1. Answer any ve: (a) Find the charge and current density that gives se to the potentials ("Ble be belt vino da (Moers Oe Sa) (0 Te sate vector J) > comeponding 1 «Haan 1) when sited 19 ne eh {ro o[() > an Hs) petty wet) >= D0 > Ts = eB) Ha tary ann upper Shy te gage usfrl Hatone b reprecned us H() = THOT (0) Ain l(a indict de Gl vine, ad he Besar eq themes Lg eter +venrvr g (Wd-e9) a «and b are constants and V(F) i «function af potion vector only. (G*) stands for tne erste of ¥ (6°) (i) Calculate the Hazitonin which corsesponcs the above Lagrangian. (d) For xed external electric and magnetic els, cleulte the rato of synchrotron los for electrons with energy 105 GrV and fr peotns with energy 7000 GeV, both moving along te same lular Path, Given, me = 0511 x 10"? GeV, my = 0058 GeV, (6) Find the mintmum energy ofan lstron to en Cen radiation in a medium with refeaetion Index 15 (0A charg is moving towards satonay ones wth wn sped» along the postive x Jaere cer sft crusted meal est iot Luci (es shown in the Br), Fie {Go Reguny afte ented cect ofan ase by te trer siting lees nate rn the charge o he mts. 2. (0) Bletromagetin i decibedyan ction $= fe ("Ru + Agit) Stow eat the trae fematin a> or A+ D(a) ymntyof he ncn. What onion dey rch pnts “peraon impo on? iv an etimation why ete td magne lds serene e Potential AP es (ep/eaat sate are xa eat at, (© lec netral ys pane cre tne dependent tuo suace cant J sich tine 20: Shor ta! Gn eared voce potanl ate heh x abow te psc a (Caewate te power radiated by unit aren of the curt earyng plane ata time t. (842) 48 4. (a) A particle of mass m and charge € approaches normally to an infinite conducting plane, {otal power radiated asm function oft eighty above the plane (6) Fora moving charge, one ean write Ena = gy (# x (@~ B) a] and = 1(¢ 6, he {rte seedleration and f ian unit vector connecting the obwerver to the charge, Ge an expression fr the angular dstbution ofthe radiated power. (c) Dra the angular distribution ofthe power dite ly lowly moving cara ‘polar pot ane 4. (a) A change ¢ f rest mass m was moving with uniform speed v (v << e) Khas been suddenly stopped after colliding to heavy scatterer, Calculate the (wavelength) spectrum of erittd teditin, (b) Action for particle of est mass m and eletec charge ¢ moving in an electromagnetic field (4") s [er nd the eatin f tn ofthe rey extrem be ton, ring te propre a in the rst frame ofthe particle and wy, ts velocity. What does the 0-th component ofthe eyvation ‘of motion signi? OH 5. (a) Consider the motion oa charg and as in an niforn magnet ld along sax tablish the equation of trajectory ae (2 ~30)*-+ (yw)? = rr bing te Larmor radon (0) The phase (6) ofthe plane electromagnetic wave i given by 5 = ot FF, where wand rare ngulae ‘eocty and ponition vector respectively. The diferent! equation tha relates tothe phase 4 gon “ 2 gi wad oy oro [ered Now suppose the ete eld for «cary poli electromagnetic wave propagating ong the dase eves ty E = BoconS) snr} the uit vector along y direction, Hence obtain the ‘Shot ofthe set of equations fry and 1c) Wee down th erent eqns concerning salar poten (Y) ae eto potenti (4 ds Cees eng cndionTnterpe! te equations of V and And subsequently # and om Speci {boar of lati unde Coulomb gage condion 33H 43) net + uy) 6.) () Using Maw’ wave equations in source fee region, establish the diferential equation for Ae Cone gauge conto ad when salar potential 20 {iy Assine A = E-Eva(t) (ex tuaig waves, establish diferent equations fr Gand ao fi) Estab the expression for tore lacromagatic energy (Hn terms o (and) Fal sow that H = 3 +s (a) Henos expres qunntition forthe standing waves with the felling expression: Hm 4h [ola + eel (v) Show that displacement cure inthe deton of «parle plate capacitor teil te em octng caren the connecting ke usteaiay +2 M. Se. (Physics) 2nd Semester Exam PHY 422 (Quantut tion 2017 ‘Mechanics 11) Answer i your o1n words as for as practicable. The marks on the right-hand margin indicate the fall marks for the question. Pull Marks: 50 - “Time: 2 Hours Answer Question No. 1 and any three from the rest 1. Answer any five questions: ae (1) Starting from the Klein-Gordon equation derive an equation of continuity. What is the dial with a probabilistic imerpretation ofthis equation? a (8) The spherical harmonics are given by [LETTE =A sme pon Vo aera) renee 1 for m £0. F(a) are sven by ve(@.e) where, €= (1) for m > O and Pie) = 1-24) (E)" Pa), where, Pi(2) is the Eth Legendre polynomial, Using thee informations, find how ¥7"(.4) transforms under space inversion (F~> —7) 4 () Show that thrce-dimensonal rotations ean be represented by SU(2) matrices. Comment about the bebiviour of this repreentation for & 2x rotation. oe (a) An electron with energy 64 eV is sattered ly a nuclear potential. The forward scattering amplitude is equal to (8-+0.5i)a, where, a9 ~ 0-5 nm isthe frst Bohr radius. Find the total ‘scattering cross section. Express your result in barn; 1 barn = 10-® mi 4 () Consider particle of mass m inthe ground state of a one dimensional infinite square well Grwidth a. Now the wall on te right Iman ale ts moved suddenly oo that the width of the wel ‘ecoms 2a. Estimate the probability that Uh particle willbe found in the ground state ofthe new expanded well 4 (f) For two identical particles write down the common eigenstates of the Hamiltonian and the (enmatation operator, Generalise the odd eigenstate to the ease of (a) three identical particles tnd (b)n identical particles. Dee 2. The quantum mechanical scattering amplitude can be written as $0.9) em | V(r) w (rar, whee, Fs the sattored wave vector, V(?) the ettering potential and (7) represents the teatteted wave function. (4) Deseribing briefly the frst Born approximation show thatthe above amplitude canbe wetten a where Kj the inet wnve yet (b) or ehatleseatering by n central potent folowing form Jhow that the Born amplitude reduces to the 10)=-2% feveenaner (6 Cats hit enantio i fom VE) = Isa) wh Yn tg oma B= J sinter. eae Conse soe dinero harmon ocator tly (+o) in its ground state is then ‘subjected to a perturbing force field of the form f(¢) = foe“®!™. (6) Using stoner perturbation theory, calelt the probably of ding the osilator in the nth excited state after the perturbation has died out (tcc). What is the maximum value of 2 as lng Bit oder ration is emer? Ye my ne (ln) = YATRA (Udo + VTE TBt). int) One my ot tt rege nel eel Houser anor of Gain (0) Draw a schmitt proba exelent st prt a function ofr. What isthe condition that this expreson of et order transition pbb vali? (©) Discus the sion ofthe lis 7-0 and 7-0 (eye 244@) (a) Defining 2, = ~iaye and eyelic permutations, whote ay ae the Dirc matrices, caleuate the commutation relations stistied by Ey, 2 (8) The Dirac wave-functons (2) and W(x!) in two inertial fames are related through ¥(2') ‘5(@)¥(2) where the Lorentz transformation is 2" = a'yz" (iv = 0,1,2,3). Sta) satis ‘SY a}y4S\a) = a7”. For an ifntesimal Lorenta transformation = 6 +, where 4, © 1. Por such an infinitesimal transformation find S(a) in terms of the matrices. (You ‘ay find the result [4, (8, Cl) = {{4,5}.C} ~ {{4,C},B} wef) 446 (a) Consider a particle of mass m and charge g moving in one dimension through the potential V(2) = aé|2|. Draw a schematic diagram af the potential with proper labeling of the turning points and obtain an estimate fr the ground state energy using the WKB quaatization rule : (0) I ag = 9991927 show that (a) 2} ad (0) ae" “Ps where 12,8 (+0)+(642) ee {b) What are the values of (a) < §,3I713,4 > / < $,-¥ity"lh,-} >, (b) < 10,0 >, CN <0 AO, weve jt tc char in dhs OS O10. > 4+ (24242) Se. gt ML ae Sem oifele. PHY 542, ie M. Se. (Physics) 3rd Semester Examination 2017 PHY 512 (Statistical Mechanics) soe YOUr oun words as far as practicable, The marks on the righthand margin indicate the full ‘marks fr the question. Macks: 0 ‘Time: 2 Hours Group A Answer any three questions (Symbols have their ust meaning) («) For sone dimensional classical harmonic oscilator with angula frequency and mass m, find the phase space volume for ergy between EA ad B's A for So (©) Foran ensemble of N identical systems, show thatthe most probable value of the number of ‘osteo having energy Bis given by whisk heancenergy-B Bee where C and 8 are constants. Obtain an expression for average eney for the system, (©) Fora canonical ensemble, prove that where the symbols have tir usual meanings. BHA) (@) Prove Liouville’ theorem fora clases ensemble of eystems (©) Consider an extreme eativitie elasical energy-momentum rlatiouship B= pe obtain the average ener. {6 The upper end of» hanging chain fixed while the lwer ends attached to & mass m. The ust links ofthe chain are elise with major axes +4 aad ninor ages! ~u and ee png fhemsclves only with either the major axis o the minor axis verte ‘The chein hes N leks oy te thermal eqiltrium at terperatre T: Find the partidon faction and ee the seen lene ofthe chain aa (a) Foran eal Ferm system at 70, obin an expeson fr Femi energy fo felons vith GRE and number density n. Show thatthe presure ls proportional to n?S, Sketch che Fee Rabun at low temperature and explain why the specific heat of the aystems sua’ ae ‘emperatures, 5 of non-interacting monoatomie molecules with Calnlate the eanonical partition function and hence DS Coder ante eystem described ina 3-diensional Hib apace, Probabilities a observe the sytem in the states (3) 4() (4) Fespectively. Find the density mats, 9, ofthe system, (orse2i+3 91/2,1/8 and 1/6 " (2) Consider a o ith corn force f. ep, e-dnensonal chal of w male which exist nto congrats & Spondng energies qa cy and lengths «and b ‘The chain ea ect to (9 Obtain the parton fonction Zy or the sat. Calculate the aveagy length a « function of f and temperature (ii) Assume that a > b and ¢o of temperature (b) Using Mayer-Ursell cluster expansion, > tg. Batimate the average length when f = 0.08 funtion show that for grand canonical real gus system Pi Sys B- ake (anes Group B ‘Answer any two questions proximation? Show that the Ising model Hamiltonian 5. (a) What do you mean by mean field apy presence ofan external magnetic fil takes the form ‘under mean eld approximation in the Fins = sadn 8) ‘where, go isthe spin of «test particle, q is the coordination number and ris the average smagnetisation. (b) Verify the result expressing temperatures as function of the average magneizationm given by the equation below for h=0. Cnttion spent P [nly the peie form of m for temperature near T= and show that >= 1/2. ‘Show that at T=, the mean fed theory for spontaneous magnetisation yds m = + 34(8+341) 6. (a) Explain the process of decimation in real space Renormalization Group (RG) theory for 8 2D quae lattice system, Tie sel factor of tranaformation is B— 2 () For one dimensional Tsing mode, show that under real space RG transformation the renor- rmalised parameters can be expressed 3s Ma = Pm cosh(2Jp + hn) cosh(2Je ~ hn) osh(2Jn +h) xsh(2Ue— he} cos(ha) cos 2Jp+ a) osh( 2 ~ he) cosh?) 46 1. (a) Starting from Langevin’ equation for Brownian motion ofa tagged pastcle, show that the ‘amplitude of fuctuations is P=2mkoT for one degree of freedom. What are the unphysical consequences ifthe dsipasion tern i ‘ot present? (Foro random varias andy then robb dsb is givenby plz) Pro (Fela) = Yea) -x(z)paly), where py(z) and pa(y) are the distributions of 2 and ¥- aed only when p(e,9) M.Se. 2™ Sem RAR —= 7 M. Se. (Physics) 2nd Semester PHY 421 (Classical Hlectrodynan Bull Marks = 5 Vino £ 2 hows “Anower QNo.I and uny hie fr the st 1. Anwer any fv of the short qustions given below, 8) BRBRIBIID rin te ates eteie Geld (5) and mati inte (2) on tn ‘fe we of Lengths [and diameter a carrying current I. Ue petal eter a> two ends of the wie ks Vy. Show by computing the Paying wt, thi epesnts Cw ever tothe wire 2b) ERBIRIBRY Let 7, be w general eank-2 tense i S-tinensonal space and bt 8) = ity + — aby) for some parameter a. Summation over epee into sis. Fi ho a a & that Sig i rnd (9 ABI The guar dependence of radiation fn change by where as the acceleration and Pi theo rai a tle of 4) RERINMEE Consider the retaivistic notin of chang pti of res a no, charge ‘electy Vand propagation vector fin the dof «pane etronngnese we, Ta un eve ‘ is constant of roton, where B= kk]. The phase of the plane wave xiven by baut-ker where w and r ore angular vocty and position veto respective. ‘The diferent ation Hh relate Co the phew is given by ee (ee 2ee- BE Fat [ECP as] Now suppose the electri eld for linypolaaed eectromagnetic mans opening aoa, the sditecion is given by E = Eajeosd,whetojie th uit meter alo y deta Olsaly the felutions ofthe st of esuations for, y and = Find abo the slatio of th st of ata fers, yand 2 when the slectie Bld changes to B= Boieosé om ‘A neutral plata placed dr ete ld (B) (ith noe ly eom FPoventsons) end magnetic ld (2) (along = section only) Shaw tha the plarlation for the poe fon fe given by a2 8 tea ah impr engl une ih mf epi a he ty ‘Foon say nese egy, Wine expen flr npn in 8) RIMES 8 sing component notation or otherwise, pune the vector identity Via) = (p- V)a+ (a Vip + px (F xa) 4x (8 x5) where and q are any two veetors in -dinesonal space 8) MEINERS Suppose there are two sets of potas, (V,A) and (VA, which eoerespond to ‘hesume deci ad magnetic feds E and B. By how mucha the two potentials fer? 8) ERBIBIIME wee down she aitzetial euations concerning salar potential (V) and sector Yeon (A) under Coulomt Gauge conan ©) EMBER opt these eguations fom pimple of essai 2) ENB sing the stove rest equntios estab inti) = = anfer THC, sa kenfen fers ©) EREIMIER Wy Coxtomt Gauge balteratly called as Transverse Gauge? 9) DEERE) we down the expression fo total power rated by noo-tlativiie dango rice (Larmos formula), From Larmors formula derive the expression of Abraham borate ecm for the radiation reaction fence 99) RRR ive tc cxc exprion os force fra dumbbell in which the total charge 8 divided soto wo halves aparated ty a fae tance Petal ish the Abraban-Lovenee ‘xprenion fr radiation rnetion fon the sbove expression of ‘Toe Lagrangian for a non-ativistic particle of mas man chargeg in an eetiomugnetc fd are the symbole have the usual mesuings. . 2) BEMIIIIR Find the eauation of motion and i ©) RIED beh tant ‘A pout particle of charge ¢ is moving with «unforen vlxity v 9), ERIBIR Wiite ene expression forthe chary 0 dewsty a8 fanetion of position ad time BRIERE ng tore anstration focl, Sdn mag ted weeny hie Note: You can ue the aes and the eign comvenlney The Lagrangian (density) of «Dirac ld i given by 2-8 8,9 — mie tere the notations have their conventional meanings, 9) EMIBNIEY his Lagroacan has « am ‘hase invariance. Write the tanafrmation in mathematical 6) EIBIRIRY i isthe Noetier current >) iz ae ‘omespoing tothe phase invariance Use the equation of motion to show that the Nother earners = bl hat the Noster eurzent Bs inded conserved asbrha M, Se. (Physics) 2nd Semester Examination 2018 PHY 432 (Quantum Mechanics 11) Anover in your oun words ax far ax practicable, The marks om the right ‘marks forthe rapetive question ri side denote the fall Fut Marks: 80, ‘Time: 2 Hours Answer Question no. Land any three frm the rest 1, Anwwor any five 4x5 (0) Consider fre pstice in» one dimensional ifnite square wll with boundaries alr = O and 7 = a, Does the Hamiltonian for this system commute with the thomentu operator” Chek excl wheter the wth energy eigenstate is al an ‘Genstate of the momentum and justify your answer. What ean you say about the patito the energy eigenstates? Give reasons in support of your answer (©) Conser the scattering by a hard sphere with radius. Find the total seatering tom ction inthe ins of igh and Tow energy ofthe ncn partic (6) A paatict iv inthe ground state of « ove-dimensiona infinite gare well with walls Ars O nnd 2 =a. At tine t= 0 the wall are suddenly removed so that the rutile bere completely fe. id the probability Tp) dp that © mentees Of the momentum a the aril will produce a result between p and p+ dp. Explain why measurement ofthe energy of the parce alter te wal ze removed need Tot give tho sane result as the itil energy (a) 26 +1 munber of noninteracting apa} pastes ar aubjced oa one dimensional Simple harmonic eeiiator (with equeney s) potential. What are the ground sate toergy and Fermi energy ofthe system at P= O°K (6) Prove for Dime matin, () Tr [Pf] = 0) Tr [>to] = 4% (0) How do anitltin () and craton (!) operator of on dimensional Barton om ‘hoe tranform under Pasty transformation (P)? Ifthe ground sate of bacmonie ‘Shilton i invariant under parity transformation, show using the party transfor eaten property of. ln) ~ (1). fn) i the n-th energy eigenstate 2, (a) Explain the essential dea behind the metho of partial waves. (b) Show that « plane wave can be expressed as a superposition between spherically CChverging and diverging waves with « well defined phase difeeace between them, {6) How isthe result of part (b) expected to be motifed in the presence of short ranged Sutra potential?” Using this idea find the expresion for the seattering amplitude and hence explain what is meant by the condition of unitarity Teae(144) ‘8. (a) Doxcrbe the easential diferences between the approaches of time independent peru Dation theory and the WKB method. (0) Use the WKB approximation to discs, the a-emiter is expected to vary with the ‘qualitative manner, bow the lifetime of ‘netic energy ofthe emitted a-paticles (c) Use the WKE quantization formula to caleulate the minimum volume occupied by a ‘quantum stat in the phase space. Stee (ate ena he! a > vin= Bae em hematite ite a ‘momentem quantum number. [Hint: Note that the derivative of the wave function i discontinuous at the boundary and the amount of discontimity i cleulabe | (b) Using Wigner-Elart theorem relate the following matrix elements as far as posible, (,,m'2}n, tam) and (fmt = wate g))n,t,m) ‘ere, jt, ate stationary states of particle bounded toa central potential. 545 1) Prov, that for Drseoqustion tobe covariant under Lorenta transformation, S94 ‘ME Here, $s the Lorents transformation for eoution forthe Dirac equation) and ‘Nis the Lorents transformation fra space time four vector. are the Dirac matrices, {2 Bog the above eat show fer a Paty tmomation,$ can be taken to be oul sy (#-+ m)and A. ~ gh (—f+-m) stisty the properties of projection at particles move in one dimension in the region 0. < 3 < L under the Sefuence of potential V such that V'= 0 inside the region and V = oo outside the region, The electrons are known tobe in a state for which = O ands = 0, What is the sround sate energy of the system ? How much does the ground state energy ahift duc a ‘very short range attractive potential V"= a2, ~ za) between these particles ? One may assume, al << $85, (©) Assuming that the Hamionian ofa system ie invariant under time reversal, prove ‘that wave fonction for non-degenerate eigenstates are real, How ean the wave fuction fora fro particle be represented by complex function &, inspite ofthe fact tha fee paticle Hamiltonian is inveriat under tte rovers? (ise Do = “ (sn Ses PMEAINCBCS) 2019 PHYSICS Module: PAY-421 (Classical Electrodynamics) oll Marks : $0 The figures inthe margin indicate fall mars Candidates ave required to give thelr answers in thir own words ‘as far as practicable. Answer amy five questions. 1. (2) Define spherical moments of charge dsuibaton, How des the numer of independent components fof spherical moments differ from Hat ofthe Caesian moments? () A charge + Qs dsributed uniformly along the = axis from 20+ a Show tat the electrostatic potential ta point 7 is given by Lair cos) mt ie freon te charges and curren, die the expression for Maxwell (6) Staring rm he eleevomagn a ‘Srettenuo nthe absence of eletrosction and magnetostricion, 4. (a) Calculate the electric and magnetic feds conesponding to the potential 4 « Kbps - 9, — 4, cee bn a conan Thr he potential sity Coulomb ge contin? ftrpet the ress (©) Show thatthe covariant equation of motion of patie nan cleewomagneti edi ate moth 2B, ea Freee Pl sia bydogen tm ashe lest at a ras equal ost Horas tine = 0, What wo aero do 0 adation? Wha the fe-span of the aH wa Ssh required tine for the aus to desea stom acording fo this moe? 4k Consider a localized source of charges an cents (confined oa very small gion of space) which ‘ay sinusoidally in time plF.1)=0(# exp ton J(F.t)=T(F)exot-iar) 4) inthe Loenz gauge provided no boundary surfaces ar preset (@) Find the veto potential 3 Find its form inthe radiation zone (kr > 1, =e) Please Tura Over @) © © 4@ © © © © 6) » © o Find the electric dipole fields & and i arising out ofthe source. Detcmine the meniar xutce ofthe power radiated and the ttal power radiated. Determine the fields E and B due o the magnetic dole moment of the sourse and same fw these ae elated tothe electric dipole feds. mS “The action ofan electromagnetic lis af) Lp Lay date ahs Establish the Maxwell's inhomogeneous equations fom this action using Lagrangi formaiom. (Gaussian unit i chosen) ‘Show that above action is invariant under gauge transformation 4, > 4, 2,2 =) povide Sour {current J is conserved and is confined in the finite par of space-time, wher a any Simi space and time Using covariant formulation, show dat he relative sereraiion of Larmoes formu So chicars velocity of charge is given by ae Argue that there are only two Lorentsinvarans in electromagnetic Seid. Which one of shee |s'8 pseudo-salar and why? Indicate clearly the arguments fr constructing complete elativic Laramie ofa chars? patie {nam elecromagnetic field. Assuming the veto pots 7 system, obtain an expression of canonical momentum 7, sagjegate to ¢. Show ur», s cmc of motion ‘Write down Faraday's equation in terms of field tensor F, ‘A sudden burst of current (7) = &S() flows through a infinite sraigheeecrically neural Determine the retarded vector potential ata distance + fom the wie ‘A charged harmonic oscillator with the manual frequency of oxilatios ey ita by aw ea impulse. The electric field produced by such an scillains charge has the fom Buby eer? Calculate the spectral width of radiation. ‘A charged panicle is subjected to a force FU) ~ KS) (for some consant & Stow thar exe {is a discontinuity in acceleration ‘Show that retarded solutions satisfy the Lorentz condition o 7 (2) Stating fom the relevant magnto-hyrodynamis equations, derive the equation : a8 oe Bl _ 2 oie(onB) nav *B Hence, cael the diffusion time and magnetic Reyno number. (©) Consider cylindrical plasma of radius with longitudinal curent. Show that mechanical pressure satisfies the relation R 1 po=s-[s ‘where symbols have their usual meaning. How does the pressure vary for constant current? (6) Indicate the mechanism of pinch effect and sources of so called sausage and kink instabilities. 3424243 2019 PHYSICS Module: PHY-422 (Quantum Mechanis-I) Pall Marks: 80 Tre fre nthe margin inet fll marks. Condes are reuied to give bir onowers in ther own words «for at proctcale Smbals hove tei ew eons Asner any fv esis. 1. (a) Apart wich iinly (¢=0) inte pound staf mnie oe dines! potent box Graseis Ova sjeded tos pemrtaton 7) = exp [sa constant] dog he Tine nerol 0. Cala up fin oder proba ing the partic in its fet exced Sat soe infor apace inan ine oe desc tential el cory egeale yao weve i) flows: =( 20 [ann (2) =(2/" sn me we is he gunn sane amis the mas of erie] (0) A puri i the pound ste of one densa mic ocr poet At #~ 0 perction Y= Ye exp) om, whe (2m (ith 3d 3 ts iin ad ces ptr Cleat po rt oder, probably hat ae sulin Teng tie (>=) the st wl ave made a Faas oy exced ne 3 2 (@) Cones yum of he sominteraing decal pia parle hatin the same spin sate [doA) ant conned me in one dnesionl infinite poral well of eg; ¥() =O for (0<20. You my ue FS nt-y) as ‘yea 4 (0) Comer oe i com meal pin ing ing nthe saeco exer mage fel. The Ess Const the tants mats a eve fo ts isan ‘ghour terson energy J. terms ‘Wher ore teractions tems} ae fn he, iemeL ren ete R= Jb sd = Jy SSrateptaesimtsay ow tat camp 3 fe ie), £)- (00) (0 19 (anh) 2)" 3.1). eee ee he congas respon {20M to ed pots? Age atthe Sten canes sas ena ae ‘ea teapots eaeoan ey te bake yes der preter (©) Lette cl exponen ad 8 be dei as: 0 H=00.M=leb © r-0,H-50,M~ya® are 1= (7-7), Mis thee pate, nd His he csi eld Now cosider «mtn witha mode egresinfor he ind fie come caring this pase watin and const apopte 10 hea ett cana 7 ued and r=o(7~T), eve ani 8 fr te weil case when 3=0. (22946) @ ee Mars s05e-¥8, r 281+ b) Ty ohih ers vii fe eg em ci be Te (9 Saw bf tn oa cn (os?)oe[r(] are] os ar where the symbols have ter usual mesing coe 7. (9 Fam he slo of be angen ation seme oben (Gere (is random fore andthe coeticent of ition) compute (9) and (Fe0)-stow that tees impo ine se wil ea an wp onsen (@) Conse the potneoes Mtoe in an iim system, denoted bythe parametr A. Let 4(y= ai) (A) Be he insane devin ofA) fo its ineindependet equi average. CW) = (SAO)A()) is the comespoding tine-caelation function. Argue tet o.a= eH) @ QW rO when 1-90 aya)

You might also like